Factor the expression.
p^2 - 10pq + 16q^2​

Answers

Answer 1

[tex]p^2 - 10pq + 16q^2=\\p^2-2pq-8pq+16q^2=\\p(p-2q)-8q(p-2q)=\\(p-8q)(p-2q)[/tex]


Related Questions

WORTH 30 POINTS PLEASE HELP!!!!! WILL GIVE POINTS

Answers

Answer:

2/3 and 4/6

Step-by-step explanation:

2/3 and 4/6.

This is because if we take 2/3 and multiply each number by 2, we get 4/6. Therefore, they are equal.

Subtract 2x^2 -9x - 7 from 8x^2 -5x + 9.

Answers

Answer:

-6x² -4x -16

Step-by-step explanation:

be watchful of signs to avoid making errors

4(x/2-2) > 2y-11 which of the following inequalities is equivalent to the inequality above?
1) 4x+2y-3 > 0
2) 4x-2y+3 > 0
3) 2x+2y-3 > 0
4) 2x-26+3 > 0

Answers

4) 2x-2y+3 > 0

although it is spelt "26" on the choices

2 cans of beans cost 98¢ how many cans can you buy for $3.92?

Answers

You can buy 8 cans for $3.92

0.98 divided by 2 = 0.49
3.92 divided by 0.49 = 8

8x0.49=3.92

Brandon is paid 150% of his regular hourly rate for overtime hours. He is paid \$45.00 an hour for overtime hoursWhat is his regular hourly rate?

Answers

Answer:

Regular hourly rate for Brandon is $30

Step-by-step explanation:

Let the payment for regular hours be $x

given that

Brandon is paid 150% of his regular hourly rate for overtime hours

payment for overtime hours = 150% of payment for regular hours

payment for overtime hours = 150/100 * x = 3x/2

Given that He is paid \$45.00 an hour for overtime hours

thus,

3x/2 = 45

=> x = 45*2/3 = 30

Thus,  regular hourly rate for Brandon is $30

football team, won 35 out of 39 games over a period of 4 years. if they keep winning pace, predict how many games you would expect them to win over the next 78 football games

Answers

Answer:

70

Step-by-step explanation:

If the team continues with same pace, they expected wins as per previous ratio:

35/39*78 = 70

Expected wins 70 out of 78 games

you decide to go to technical school for 2 years to get a degreee in computer networking. it takes you 8 quarters to complete the program abd it costs $830 per quarter for tuition in addition to books for the programs that cost 1800 how much will is cost you to get your degree
whats the answer ​

Answers

9514 1404 393

Answer:

  $8440

Step-by-step explanation:

8 quarters at $830 per quarter is 8×$830 = $6640.

When the cost of books is added, the total is ...

  $6640 +1800 = $8440

Solve this inequality: 4x-8>-40

Answers

Answer:

x > - 8

Step-by-step explanation:

4x - 8 > - 40

4x > - 40 + 8

4x > - 32

Divide 4 on both sides,

4x / 4 > - 32 / 4

x > - 8

Answer:

x > 12

Step-by-step explanation:

4x - 8 > 40

Add 8 to both sides

4x > 48

Divide both sides by 4

x > 12

find the area of the triangle and round to the nearest tenth

Answers

Answer: 73.5mi²

Step-by-step explanation:

Area of triangle = 1/2×b×h

= 1/2×14×10.5

= 7×10.5

=73.5mi²

please click thanks and mark brainliest if you like :)

A basketball player made 5 baskets during a game. Each basket was worth either 2 or 3 points. How many different numbers could represent the total points scored by the player?

Answers

The lowest he will get is 10 points and highest 15, 3-2=1 so minimum increment is 1. Count 10 to 15 by increment of 1 you will have 6 different numbers.

If the average fixed cost (AFC) of producing 5 bags of rice is $20.00, the average fixed cost of producing 10 bags will be​

Answers

Answer:$40.00

Step-by-step explanation:first divide 20 by 5 and the answer will be 4. now multiply 10 into 4 and you'll get the answer $40.00

Diana get a gift card with a value of $55, and her favorite drink cost $2.20. How many plain black coffee can she buy with the gift card

Answers

Answer:

Twouufyjughgyuioiu567uhu888

Answer:

55/2.20=25 so 25 black coffees

Step-by-step explanation:

eastern Aviation equipment pays bob Coleman a $1310 monthly salary plus a 12% commission on merchandise he sells each month. assume Bob's sales were $76,200 for the last month amount of commission: gross pay:​

Answers

Answer:

12/100 *76200 =$9144

so gross pay = $1310 +$9144 =$10454

List the angles in order from the largest to the smallest for ABC.
AB= 14, AC = 15, BC = 16

Answers

Answer:

B. ∠A, ∠B, ∠C

Step-by-step explanation:

1. Draw a model with AB as the shortest line and BC as the longest line.

∠A connects the two shortest lines, making it the largest angle.

∠B connects the shortest and the longest lines, making it the second largest angle.

∠C connects the two longest lines, making it the smallest angle.

Answer:

A > B > C

Step-by-step explanation:

Ypu probably wouldn't think about it unless someone pointed it out, but if you look at a triangle of any type you can see that the sizes of the sides are directly related to the sizes of the angles opposed to them.

By this I mean, the largest side will have the largest angle across from it and the smallest side will have the smallest angle.

Based off of my drawing, it looks like the order is angle A, then B, then, and then C.

The sum of four
consecutive odd number is 8o. Find the number​

Answers

Answer:

The sum of 4 consecutive odd number is 80

Let X be the first of these numbers

Then the next odd number is X+2

The third is X+4The fourth is X+6

All of these add up to 80

(X) + (X+2) + (X+4) + (X+6) = 80

Using the commutative and associative laws, let's transform this equation into

(X + X + X + X) + (2 + 4 + 6) = 804X + 12 = 80

Subtract 12 from both sides of the equation gives4X = 68

Divide both sides by 4 gives

X = 17

Going back to the original question:What are the 4 consecutive odd numbers: 17, 19, 21, 23Checking our answer:17 + 19 + 21 + 23 = 80 Correct!

Find all complex solutions of 2x^2+x+6=0. (If there is more than one solution, separate them with commas.)

Answers

Answer:

x=-1/4+i*sqrt(47)/4 and x=x=-1/4-i*sqrt(47)/4

Step-by-step explanation:

Using quadratic formula, x=(-1±sqrt(1-48))/4.

x=-1/4+i*sqrt(47)/4 and x=x=-1/4-i*sqrt(47)/4

Answer:

If you do not understand any steps, please feel free to comment down below.

1. Solve the system of equations. y = –3x + 4 x + 4y = –6 A. x = –2,y = –1 B. x = –2,y = 10 C. x = 2,y = –2 D. x = 3,y = –5 E. x = 4,y = –8

Answers

Answer:

C. x = 2, y = -2

Step-by-step explanation:

y = -3x + 4

x + 4y = -6

x + 4(-3x + 4) = -6

x - 12x + 16 = -6

-11x = -22

x = 2

y = -3(2) + 4 = -2

What is the slope-intercept equation of the line below?



10 minutes left

Answers

Answer:

y=-3x+4

Step-by-step explanation:

The y intercept is 4 because the line crosses the y axis at the 4 tic mark

The slope will be -3 because the y decreases by 3 every time the x incerases by 1

y=mx+b

y=-3x+4

If 2x + 5 = 8x, then 12x = ?
A 5
B
10
C
15
D 20

Answers

Answer:

10

Step-by-step explanation:

2x + 5 = 8x

Subtract 2x from each side

2x-2x + 5 = 8x-2x

5 = 6x

We want 12x so multiply each side by 2

2*5 = 6x*2

10 = 12x

Answer:

B. 10

Step-by-step explanation:

To find 12x, you first need to find the value of x using the first equation:

[tex]2x+5=8x[/tex]

You need to get the variables (x) on the same side of the equation in order to simplify them. To do this, use reverse operations. Subtract 2x from both sides to keep the equation balanced:

[tex]2x-2x+5=8x-2x\\\\5=6x[/tex]

Now isolate the variable (x) by dividing both sides of the equation by 6 (using reverse operations):

[tex]\frac{5}{6}=\frac{6x}{6} \\\\\frac{5}{6}=x[/tex]

Now insert the given value of x into 12x:

[tex]12(\frac{5}{6})[/tex]

Simplify:

[tex]12*\frac{5}{6} \\\\\frac{12}{1}*\frac{5}{6}\\\\\frac{60}{6}=10[/tex]

12x equals 10.

:Done

Find the area of the figure.

A =

Is it m, m2, or m3

Answers

Answer:

348 m^2

Step-by-step explanation:

The figure is made up of a rectangle 24 m by 12 m, and a triangle with a 24 m base and a 5 m height.

A = LW + bh/2

A = 24 m * 12 m + (24 m)(5 m)/2

A = 288 m^2 + 60 m^2

A = 348 m^2

How many terms of the series 2 + 5 + 8 + … must be taken if their sum is 155

Answers

9514 1404 393

Answer:

  10

Step-by-step explanation:

The sum of terms of an arithmetic series is ...

  Sn = (2a +d(n -1))·n/2 = (2an +dn^2 -dn)/2

For the series with first term 2 and common difference 3, the sum is 155 for n terms, where ...

  155 = (3n^2 +n(2·2 -3))/2

Multiplying by 2, we have ...

  3n^2 +n -310 = 0 . . . . . arranged in standard form

Using the quadratic formula, the positive solution is ...

  n = (-1 +√(1 -4(3)(-310)))/(2(3)) = (-1 +√3721)/6 = (61 -1)/6 = 10

10 terms of the series will have a sum of 155.

Answer: 10 terms

Step-by-step explanation:

[tex]\displaystyle \ \Large \boldsymbol{} S_n=\frac{2a_1+d(n-1)}{2} \cdot n =155 \\\\ \frac{4+3(n-1)}{2} \cdot n =155 \\\\\\ 4n+3n^2-3n=310 \\\\ 3n^2+n-310=0 \\\\D=1+3720=3721=61^2\\\\n_1=\frac{61-1}{6} =\boxed{10} \\\\\\n_2=\frac{-61-1}{3} \ \ \o[/tex]

A cupboard costing Rs.16800 is depreciated at the rate of 15% per year. What will be the cost of the cupboard after 2 years.​

Answers

Answer:

Rs. 11132

Step-by-step explanation:

formula for depriciation= P(1-R/100)^T

p= principal, r=rate, t=time

Answer:

15162

Step-by-step explanation:

after 1 year= 16800*95%= 15960

after 2 year= 15960*95%= 15162

xThe closing price of Schnur Sporting Goods Inc. common stock is uniformly distributed between $18 and $28 per share. What is the probability that the stock price will be: More than $20? (Round your answer to 4 decimal places.)

Answers

Answer:

The  probability is [tex]P(X > 20 ) = 0.8[/tex]

Step-by-step explanation:

From the question we are told that

  The closing price of Schnur Sporting Goods Inc. common stock is uniformly distributed between $18 and $28 per share.

Given that the stock is uniformly distributed then the probability that the stock price will be more than $20 is mathematically evaluated as

       [tex]P(X > 20 ) = 1 - P(X < 20 )[/tex]

Since it is uniformly distribute  between $18 and $28 per share then we can solve is as follows

=>     [tex]P(X > 20 ) = 1 - [\frac{ 20 - 18 }{28 -18} ][/tex]

=>    [tex]P(X > 20 ) = 0.8[/tex]

A person has decided to construct a frequency distribution for a set of data containing 100 numbers. The lowest number is 20 and the highest number is 80. If 5 classes are used, the class width should be approximately _______.

Answers

Answer:

The answer is "12".

Step-by-step explanation:

Lowest number[tex]=20\\\\[/tex]

highest number[tex]=80\\\\[/tex]

Number of classes[tex]=5\\\\[/tex]

Range=highest number-lowest number

          [tex]=80-20\\\\=60\\\\[/tex]

class width [tex]=\frac{Range}{\text{number of classes}}[/tex]

                   [tex]=\frac{60}{5}\\\\=12[/tex]

A simple random sample of 60 households in city 1 is taken. In the sample, there are 45 households that decorate their houses with lights for the holidays. A simple random sample of 50 households is also taken from the neighboring city 2. In the sample, there are 40 households that decorate their houses. What is a 95% confidence interval for the difference in population proportions of households that decorate their houses with lights for the holidays

Answers

Answer:

The calculated value of z = - 0.197  falls in the critical region therefore we reject the null hypothesis and conclude that  at  the 5% significance level there is significant difference in population proportions of households that decorate their houses with lights for the holidays

Step-by-step explanation:

We formulate the null and alternative hypotheses as

H0: p1= p2 there is no difference in population proportions of households that decorate their houses with lights for the holidays

against Ha : p1≠ p2  (claim)  ( two sided)

The significance level is set at ∝= 0.05

The critical value for two tailed test at alpha=0.05 is ± 1.96

or Z∝= 0.05/2= ± 1.96

The test statistic is

Z = p1-p2/√pq(1/n1 +1/n2)

p1= proportions of households  decorating in city 1  = 45/60=0.75

p2= proportions of households  decorating in city 2 = 40/50= 0.8

p = the common proportion on the assumption that the two proportion are same.

p =   [tex]\frac{n_1p_1 +n_2p_2}{n_1+n_2}[/tex]

Calculating

p =60 (0.75) + 50 (0.8) / 110

p=  45+ 40/110= 85/110 = 0.772

so  q = 1-p= 1- 0.772= 0.227

Putting the values in the test statistic and calculating

z= 0.75- 0.8/ √0.772*0.227( 1/60 + 1/50)

z= -0.05/√ 0.175244 ( 110/300)

z= -0.05/0.25348

z= -0.197

The calculated value of z = - 0.197  falls in the critical region therefore we reject the null hypothesis and conclude that  at  the 5% significance level there is significant difference in population proportions of households that decorate their houses with lights for the holidays

Joseph is 33 years old. Five years ago, He was twice as old as Ann. How old will Ann be in 5 years time?

Answers

Answer:

19 years old

Step-by-step explanation:

[tex]Joseph = 33 \:years \:old\\ \\Let \: ann's \:age be x\\\\33-5 = 2x\\\\28 = 2x\\\\Divide \:both \:sides \:of \:the \:equation \: by \:2\\\\\frac{2x}{2} = \frac{28}{2} \\\\x = 14\\\\Ann's \:present \:age \:= 14\\\\In \:5 \:years \:time ; \\\\14+5 = 19\\[/tex]

If In (x) = 3.53, what is the value of x ?

Answers

I need more information please ?

A human factor expert recommends that there be atleast 9 square ft of floor space in a classroom for every student in the class. Find the min space required for 49 students

Answers

I think the answer is 441 ft I’m sorry if it’s wrong

use the diagram to answer the question. AB corresponds to which line segment?

Answers

Answer:

DE

Step-by-step explanation:

I hope this helps!

Determine the decision criterion for rejecting the null hypothesis in the given hypothesis​ test; i.e., describe the values of the test statistic that would result in rejection of the null hypothesis. Suppose you wish to test the claim that ​, the mean value of the differences d for a population of paired​ data, is greater than 0. Given a sample of n15 and a significance level of ​0.01, what criterion would be used for rejecting the null​ hypothesis?

Answers

Answer:

reject null hypothesis if calculated t value > 2.624

Step-by-step explanation:

n = 15

To calculate degree of freedom, n -1 = 14

The claim says ud>0

The decision rule would be to reject this null hypothesis if the test statistics turns out to be greater than the critical value.

With df =14

Confidence level = 0.01

Critical value = 2.624 (for a one tailed test)

If the t value calculated is > 2.624, we reject null hypothesis.

Using the t-distribution and it's critical values, the decision rule is:

t < 2.624: Do not reject the null hypothesis.t > 2.624: Reject the null hypothesis.

At the null hypothesis, we test if the mean is not greater than 0, that is:

[tex]H_0: \mu \leq 0[/tex]

At the alternative hypothesis, we test if the mean is greater than 0, that is:

[tex]H_1: \mu > 0[/tex].

We then have to find the critical value for a right-tailed test(test if the mean is more than a value), with 15 - 1 = 14 df and a significance level of 0.01. Using a t-distribution calculator, it is [tex]t^{\ast} = 2.624[/tex].

Hence, the decision rule is, according to the test statistic t:

t < 2.624: Do not reject the null hypothesis.t > 2.624: Reject the null hypothesis.

A similar problem is given at https://brainly.com/question/13949450

Other Questions
-5x-10=10 solve for x in the krebs cycle, a. pyruvate is broken down into Co2 moleculesb. glucose is broken down into NADH and CO2c. Co2 is concerted in pyruvate and O2d. NADH and CO2 are converted into ATP Given the following balanced reaction: 2Na(s) + F2(g) --> 2NaF(s)a) How many moles of NaF will be made from 2.6 moles of F2?b) How many moles of NaF will be made from 4.8 moles of Na? The standard normal curve shown below models the population distribution of a random variable. What proportion of the values in the population does not lie between the two z-scores indicated on the diagram? z = -1.2 z = 0.85 Fill in the blank in the following sentence with the appropriate preterite verbbelow.Yoa la playa con mi familia el verano pasado.O A. voy B. fuiO C. fueO D. soy What is the word that describes Americas relationship with China and the Soviet Union, as fostered by Nixon and Kissinger? Which statement about modern Israel is most accurate?O It is made up only of Jewish people.O It is the homeland for Jewish people.It has been a Jewish holy site for fifty years.O It features modern beliefs and rejects the past. In the course of selling a home to a buyer, a broker told the buyer that the home's foundation was "solid as a rock", when he knew for a fact that it was slowly sinking into the landfill on which it was built. In this situation, the broker's conduct would be BEST defined as a Is {3,} a defined set Why does the DNA that makes up the chromosomes need to replicate before the start of meiosis? Jonah will cover a cube in wrapping paper. Each edge of the cube is 25 cm long. What is the least amount ofwrapping paper he needs to cover the cube?15 625 square centimeters25 square centimeters37.5 square centimeters42 25 square centimetersSave and ExitNextSubmMO Tempe Office Services and Supplies (TOSS) provides various products and services in the Tempe Research Park, home to numerous high-tech and bio-tech companies. Making color copies is one of its most popular and profitable services. The controller performed a regression analysis of data from the Color Copy Department with the following results:Intercept 238.6R square 0.968Number of observation 6X coefficient 0.069Required:What is the variable cost per color copy for TOSS? PLEASE HELP ill give brainliest to best and correct answer QUESTION 3Individuals will experience positive or negative stress as a natural part of life.TrueFalse Which of the following is a major agricultural export of East Africa?Question 2 options:corncocoacoffeesugar cane The approach to measuring GDP that adds together compensation to employees, rents, interest income, dividends, undistributed corporate profits, proprietors' income, indirect business taxes, the consumption of fixed capital, and net foreign factor income earned in the United States is the: 19.The horses swim in the river to avoid danger. Which words make up the ENTIRE infinitive phrase in this sentence? Find the missing side. Round answer to the nearest tenth. Most reactions, including enzyme-catalyzed reactions, proceed faster at higher temperatures. However, for a given enzyme, the rate drops off abruptly at a certain temperature. Account for this behavior. Greek scientists and thinkers were able to make advances in science because they considered ___________ events the result of natural causes. unnatural Hellenistic phalanx natural